Вверх
Вариант 7
1. В инерциальной системе отсчёта сила 50 Н сообщает телу массой 5 кг некоторое ускорение. Какая сила сообщает такое же ускорение телу массой 3 кг?

Ответ: ________________ Н.

Верный ответ: 30

P.S. Нашли ошибку в задании? Пожалуйста, сообщите о вашей находке ;)
При обращении указывайте id этого вопроса - 15921.

2. На тело массой 2,5 кг, движущееся прямолинейно в инерциальной системе отсчёта, в течение 5 с в направлении движения действует постоянная сила, при этом импульс тела увеличивается на 40 кг ⋅ м/с. Какова величина силы, действующей на тело?

Ответ: ________________ Н.

Верный ответ: 8

P.S. Нашли ошибку в задании? Пожалуйста, сообщите о вашей находке ;)
При обращении указывайте id этого вопроса - 15682.

3. Отношение импульса легкового автомобиля к импульсу мотоцикла \( \frac{{{p_1}}}{{{p_2}}} \) = 5. Каково отношение их скоростей \( \frac{{{\upsilon _1}}}{{{\upsilon _2}}} \), если отношение массы легкового автомобиля к массе мотоцикла \( \frac{{{m_1}}}{{{m_2}}} \) = 2,5?

Ответ: ________________ .

Верный ответ: 2

P.S. Нашли ошибку в задании? Пожалуйста, сообщите о вашей находке ;)
При обращении указывайте id этого вопроса - 21409.

4. Какова частота звуковых волн в среде, если скорость звука в этой среде равна 1000 м/с, а период колебаний составляет 2 мс?

Ответ: ________________ Гц.

Верный ответ: 500

P.S. Нашли ошибку в задании? Пожалуйста, сообщите о вашей находке ;)
При обращении указывайте id этого вопроса - 21410.

5. Два одинаковых вертикальных сообщающихся цилиндрических сосуда заполнены водой и закрыты поршнями массами M1 = 1 кг и M2 = 2 кг. Когда система находится в равновесии, правый поршень с площадью основания 100 см2 расположен ниже левого на величину h = 10 см.
Задание ЕГЭ по физике
Выберите из предложенного перечня все верные утверждения.

Верный ответ: 14

P.S. Нашли ошибку в задании? Пожалуйста, сообщите о вашей находке ;)
При обращении указывайте id этого вопроса - 15534.

6. На поверхности воды плавает брусок из древесины плотностью 800 кг/м3. Брусок заменили на другой брусок той же массы и с той же площадью основания, но из древесины плотностью 500 кг/м3. Как при этом изменились масса вытесненной воды и действующая на брусок сила Архимеда?

Для каждой величины определите соответствующий характер изменения:

1) не изменилась
2) уменьшилась
3) увеличилась

Запишите в таблицу выбранные цифры для каждой физической величины. Цифры в ответе могут повторяться.
Задание ЕГЭ по физике

Верный ответ: 11

P.S. Нашли ошибку в задании? Пожалуйста, сообщите о вашей находке ;)
При обращении указывайте id этого вопроса - 15445.

7. При уменьшении абсолютной температуры средняя кинетическая энергия хаотического теплового движения молекул неона уменьшилась в 3 раза. Конечная температура газа равна 300 К. Какова начальная температура газа?

Ответ: ________________ К.

Верный ответ: 900

P.S. Нашли ошибку в задании? Пожалуйста, сообщите о вашей находке ;)
При обращении указывайте id этого вопроса - 15657.

8. На рисунке показано, как менялось давление газа в зависимости от его объёма при переходе из состояния 1 в состояние 2, а затем в состояние 3. Каково отношение работ газа \( \frac{{{A_{12}}}}{{{A_{23}}}} \) в этих двух процессах?
Задание ЕГЭ по физике

Верный ответ: 0,75

P.S. Нашли ошибку в задании? Пожалуйста, сообщите о вашей находке ;)
При обращении указывайте id этого вопроса - 15418.

9. На pV-диаграмме показаны два процесса, проведённые с одним и тем же постоянным количеством газообразного неона.
Из приведённого ниже списка выберите все верные утверждения, характеризующие процессы на графике.
Задание ЕГЭ по физике

Верный ответ: 34

P.S. Нашли ошибку в задании? Пожалуйста, сообщите о вашей находке ;)
При обращении указывайте id этого вопроса - 15480.

10. Один моль одноатомного идеального газа участвует в процессе 1-2-3, график которого изображён на рисунке в координатах p-T, где p - давление газа, T - абсолютная температура газа. Как изменяются объём газа V в ходе процесса 1-2 и плотность газа ρ в ходе процесса 2-3?
Задание ЕГЭ по физике
Для каждой величины определите соответствующий характер изменения:

1) не изменяется
2) уменьшается
3) увеличивается

Запишите в таблицу выбранные цифры для каждой физической величины. Цифры в ответе могут повторяться.
Задание ЕГЭ по физике

Верный ответ: 13

P.S. Нашли ошибку в задании? Пожалуйста, сообщите о вашей находке ;)
При обращении указывайте id этого вопроса - 15841.

11. Силы электростатического взаимодействия между двумя неподвижными точечными заряженными телами равны по модулю 20 мН. Каким станет модуль этих сил, если заряд каждого тела увеличить в 3 раза?

Ответ: ________________ мН.

Верный ответ: 180

P.S. Нашли ошибку в задании? Пожалуйста, сообщите о вашей находке ;)
При обращении указывайте id этого вопроса - 15692.

12. Две частицы с одинаковыми массами и зарядами q и 2q влетают в однородное магнитное поле перпендикулярно вектору магнитной индукции со скоростями \( \upsilon \) и \( 2\upsilon \) соответственно. Определите отношение модулей сил \( \frac{{{F_1}}}{{{F_2}}} \), действующих на них со стороны магнитного поля.

Ответ: ________________ .

Верный ответ: 0,25

P.S. Нашли ошибку в задании? Пожалуйста, сообщите о вашей находке ;)
При обращении указывайте id этого вопроса - 21411.

13. Энергия магнитного поля катушки с током равна 0,64 Дж. Индуктивность катушки равна 20 мГн. Какова сила тока в катушке?

Ответ: ________________ А.

Верный ответ: 8

P.S. Нашли ошибку в задании? Пожалуйста, сообщите о вашей находке ;)
При обращении указывайте id этого вопроса - 15814.

14. Проволочная рамка площадью 30 см2 помещена в однородное магнитное поле. Проекция Bn индукции магнитного поля на нормаль к плоскости рамки изменяется во времени t согласно графику на рисунке.
Задание ЕГЭ по физике
Из приведённого ниже списка выберите все верные утверждения о процессах, происходящих в рамке.

Верный ответ: 14

P.S. Нашли ошибку в задании? Пожалуйста, сообщите о вашей находке ;)
При обращении указывайте id этого вопроса - 15695.

15. Неразветвлённая электрическая цепь постоянного тока состоит из источника тока и подключённого к его выводам резистора. Как изменятся при уменьшении сопротивления резистора тепловая мощность, выделяющаяся на внутреннем сопротивлении источника, и электродвижущая сила источника?

Для каждой величины определите соответствующий характер изменения:

1) не изменяется
2) уменьшается
3) увеличивается

Запишите в таблицу выбранные цифры для каждой физической величины. Цифры в ответе могут повторяться.
Задание ЕГЭ по физике

Верный ответ: 31

P.S. Нашли ошибку в задании? Пожалуйста, сообщите о вашей находке ;)
При обращении указывайте id этого вопроса - 15576.

16. Закон радиоактивного распада ядер некоторого изотопа имеет вид \( N = {N_0} \cdot {2^{ - \lambda t}} \), где \( \lambda \) = 0,2 с-1. Каков период полураспада ядер?

Ответ: ________________ с.

Верный ответ: 5

P.S. Нашли ошибку в задании? Пожалуйста, сообщите о вашей находке ;)
При обращении указывайте id этого вопроса - 15518.

17. На рисунке изображена упрощённая диаграмма нижних энергетических уровней атома. Нумерованными стрелками отмечены некоторые возможные переходы атома между этими уровнями. Какой из этих четырёх переходов связан с поглощением света наименьшей частоты, а какой с излучением света наибольшей частоты?
Задание ЕГЭ по физике
Установите соответствие между процессами поглощения и испускания света и стрелками, указывающими энергетические переходы атома.

К каждой позиции первого столбца подберите соответствующую позицию из второго столбца и запишите в таблицу выбранные цифры под соответствующими буквами.
ПРОЦЕССЫ

А) излучение света наибольшей частоты
Б) поглощение света наименьшей частоты
ЭНЕРГЕТИЧЕСКИЕ ПЕРЕХОДЫ

1) 1
2) 2
3) 3
4) 4

Верный ответ: 23

P.S. Нашли ошибку в задании? Пожалуйста, сообщите о вашей находке ;)
При обращении указывайте id этого вопроса - 15819.

18. Выберите все верные утверждения о физических явлениях, величинах и закономерностях. Запишите цифры, под которыми они указаны.

Верный ответ: 345

P.S. Нашли ошибку в задании? Пожалуйста, сообщите о вашей находке ;)
При обращении указывайте id этого вопроса - 15649.

19. В журнале 50 листов. По результатам измерения с помощью линейки толщина журнала составляет 0,9 см. Чему равна толщина одного листа по результатам этих измерений, если погрешность линейки равна ±1 мм?

Ответ: (________ ± ________) мм.

Верный ответ: 0,180,02

P.S. Нашли ошибку в задании? Пожалуйста, сообщите о вашей находке ;)
При обращении указывайте id этого вопроса - 15580.

20. Необходимо экспериментально изучить зависимость силы Архимеда, действующей на тело, погружённое в жидкость, от объёма погружённого тела.
Какие две установки следует использовать для проведения такого исследования?
Задание ЕГЭ по физике

Верный ответ: 14

P.S. Нашли ошибку в задании? Пожалуйста, сообщите о вашей находке ;)
При обращении указывайте id этого вопроса - 15851.

Часть 2

21. Многовитковая катушка медного провода подключена к источнику тока через реостат. Вблизи торца катушки на шёлковых нитях подвешено замкнутое медное кольцо с малым сопротивлением. Ось кольца совпадает с осью катушки (см. рисунок). Опишите, как начнёт двигаться кольцо (притянется, оттолкнётся или останется неподвижным относительно катушки), если движок реостата резко сдвинуть вверх в крайнее положение. Ответ поясните, указав, какие физические явления и закономерности Вы использовали для объяснения.
Задание ЕГЭ по физике

1. Кольцо оттолкнётся от катушки.
2. При смещении движка реостата вверх его сопротивление уменьшается до нуля, а сила тока в катушке согласно закону Ома для полной цепи увеличивается \( \left( {I = \frac{\varepsilon }{{R + r}}} \right) \). При этом увеличивается поток вектора магнитной индукции через кольцо. По закону электромагнитной индукции в кольце возникает ЭДС индукции, появляется индукционный ток. В соответствии с правилом Ленца вектор магнитной индукции поля индукционного тока будет направлен против вектора магнитной индукции поля катушки. Взаимодействие токов в кольце и катушке приводит к тому, что кольцо отталкивается от катушки влево - в область, где магнитное поле катушки слабее, чем у торца катушки.

Ответ: кольцо отталкивается от катушки влево.

Примечание для экспертов.
Магнитные свойства выражены у меди слабо, индукционный ток в неподвижном кольце вблизи катушки с постоянным током равен нулю, поэтому сначала, пока движок реостата находится посередине, катушка практически не воздействует на медное кольцо и оно остаётся неподвижным.
В состоянии максимального отклонения влево кольцо не находится в равновесии, поэтому оно начинает возвращаться в исходное положение. При этом движении в кольце снова возникает индукционный ток, который замедляет движение и препятствует возможным колебаниям кольца на нитях.
В результате кольцо возвращается в исходное положение равновесия и остаётся в нём неподвижным.
(Эти рассуждения для полного верного ответа не требуются.)

P.S. Нашли ошибку в задании? Пожалуйста, сообщите о вашей находке ;)
При обращении указывайте id этого вопроса - 15702.

22. Груз массой 200 г подвешен на пружине жёсткостью 100 Н/м к потолку лифта. Лифт равноускоренно движется вниз, набирая скорость. Каково ускорение лифта, если удлинение пружины постоянно и равно 1,5 см?

1. Выберем инерциальную систему отсчёта «шахта лифта», направив вертикальную ось \( Oy \) вниз по ускорению и расставив силы, действующие на груз, как показано на рисунке.

Задание ЕГЭ по физике
2. Запишем II закон Ньютона для груза в выбранной ИСО в проекциях на ось \( Oy \):

\( Oy:mg - {F_{упр}} = ma \)

откуда \( a = g - \frac{{{F_{упр}}}}{m} \). (2)

3. По закону Гука модуль силы упругости: \( {F_{упр}} = k\Delta y \), (3)
где \( k \) — жёсткость, а \( \Delta y \) — заданное удлинение пружины.

4. Подставив (3) в (2), находим проекцию искомого ускорения:

\( a = g - \frac{{k\Delta y}}{m} \)\( = 10 - \frac{{100 \cdot 0,015}}{{0,2}} \) = 2,5 м/с2

Ответ: a = 2,5 м/с2

P.S. Нашли ошибку в задании? Пожалуйста, сообщите о вашей находке ;)
При обращении указывайте id этого вопроса - 21412.

23. Пластины большого по размерам плоского конденсатора расположены горизонтально на расстоянии d = 2 см друг от друга. Напряжение на пластинах конденсатора 10 кВ. В пространстве между пластинами падает капля жидкости. Масса капли m = 5 мг. При каком значении заряда q капли её скорость будет постоянной? Влиянием воздуха на движение капли пренебречь.
Задание ЕГЭ по физике

Скорость капли постоянна, если равнодействующая сил, действующих на каплю, равна нулю. На каплю действуют сила тяжести, направленная вниз, и сила со стороны электростатического поля, которая для постоянства скорости должна быть направлена вверх. Следовательно, капля заряжена отрицательно. Для того чтобы капля двигалась с постоянной скоростью, эти силы должны быть равны по модулю: \( mg = \left| q \right|E \).

Напряжённость однородного электростатического поля конденсатора связана с напряжением между пластинами соотношением \( E = \frac{U}{d} \).

Следовательно, заряд капли

\( q = - \frac{{mdg}}{U} = \)\( \frac{{5 \cdot {{10}^{ - 6}} \cdot 2 \cdot {{10}^{ - 2}} \cdot 10}}{{{{10}^4}}} = - {10^{ - 10}} \) Кл

Ответ: q = -10-10 Кл

P.S. Нашли ошибку в задании? Пожалуйста, сообщите о вашей находке ;)
При обращении указывайте id этого вопроса - 21413.

24. В закрытом сосуде объёмом V = 10 л находится влажный воздух массой m = 18 г при температуре t = 80 °С и давлении p = 2 · 105 Па. Определите массу паров воды в сосуде.

Влажный воздух представляет собой смесь паров воды и сухого воздуха, следовательно, \( m = {m_В} + {m_П} \), где \( m \), \( {m_В} \), \( {m_П} \), — масса влажного воздуха, сухого воздуха и водяного пара соответственно. Согласно закону Дальтона, \( p = {p_В} + {p_П} \), где \( p \), \( {p_В} \), \( {p_П} \), — давление влажного воздуха, парциальное давление сухого воздуха и парциальное давление водяного пара соответственно.

Выразим из уравнения состояния идеального газа \( pV = \frac{m}{\mu }RT \) парциальное давление пара \( {p_П} = \frac{{{m_П}RT}}{{{\mu _2}V}} \) и сухого воздуха \( {p_В} = \frac{{(m - {m_П})RT}}{{{\mu _1}V}} \), где \( {{\mu _1}} \) — молярная масса сухого воздуха, \( {{\mu _2}} \) — молярная масса водяного пара.

Получаем \( p = \left\{ {\left. {\frac{m}{{{\mu _1}}} + \frac{{{m_П}}}{{{\mu _1}}}\left( {\frac{{{\mu _1}}}{{{\mu _2}}} - 1} \right)} \right\}} \right. \cdot \frac{{RT}}{V} \), откуда

\( {m_П} = \frac{{\frac{{pV{\mu _1}}}{{RT}} - m}}{{\frac{{{\mu _1}}}{{{\mu _2}}} - 1}} = \)\( \frac{{\frac{{2 \cdot {{10}^5} \cdot {{10}^{ - 2}} \cdot 0,029}}{{8,31 \cdot 353}} - 0,018}}{{\frac{{0,029}}{{0,018}} - 1}} \) ≈ 2,9 · 10-3 кг = 2,9 г

Ответ: mП ≈ 2,9 г

P.S. Нашли ошибку в задании? Пожалуйста, сообщите о вашей находке ;)
При обращении указывайте id этого вопроса - 21414.

25. Горизонтальный проводящий стержень прямоугольного сечения поступательно движется с ускорением a = 1,5 м/с2 вверх по гладкой диэлектрической наклонной плоскости в вертикальном однородном магнитном поле (см. рисунок). Угол наклона плоскости \( \alpha \) = 30°. Отношение массы стержня к его длине \( \frac{m}{L} \) = 0,1 кг/м. Модуль индукции магнитного поля B = 0,3 Тл. Определите силу тока \( I \), протекающего по стержню. Сделайте рисунок с указанием сил, действующих на стержень.
Задание ЕГЭ по физике

Задание ЕГЭ по физике
1. На рисунке показаны силы, действующие на стержень с током:
— сила тяжести \( m\vec g \), направленная вертикально вниз
— сила реакции опоры \( {\vec N} \), направленная перпендикулярно к наклонной плоскости
— сила Ампера \( {{\vec F}_A} \), направленная горизонтально вправо, что вытекает из правила левой руки

2. Модуль силы Ампера \( {F_A} = IBL \), (1)
где \( L \) — длина стержня.

3. Систему отсчёта, связанную с наклонной плоскостью, считаем инерциальной. Запишем второй закон Ньютона в проекциях на ось x (см. рисунок):

\( m{a_x} = - mg\sin \alpha + IBL\cos \alpha \), (2)
где m — масса стержня.

Отсюда: \( I = \frac{m}{L} \cdot \frac{{{a_x} + g\sin \alpha }}{{B\cos \alpha }} = \)\( 0,1 \cdot \frac{{1,5 + 10 \cdot \sin 30^\circ }}{{0,3 \cdot \cos 30^\circ }} \) ≈ 2,5 А (3)

Ответ: \( I \) ≈ 2,5 А

P.S. Нашли ошибку в задании? Пожалуйста, сообщите о вашей находке ;)
При обращении указывайте id этого вопроса - 21415.

26. Груз массой M = 800 г соединён невесомой и нерастяжимой нитью, перекинутой через гладкий невесомый блок, с бруском массой m = 400 г. К этому бруску на лёгкой пружине с жёсткостью k = 80 Н/м подвешен второй такой же брусок. т Длина нерастянутой пружины \(l \) = 10 см, коэффициент трения груза о поверхность стола \(\mu \) = 0,2. Определите длину пружины при движении брусков, считая, что при этом движении она т постоянна. Сделайте рисунок с указанием сил, действующих на тела. Обоснуйте применимость используемых законов к решению задачи.
Задание ЕГЭ по физике

Обоснование
Задачу будем решать в инерциальной системе отсчёта, связанной с поверхностью стола. Будем применять для грузов и бруска законы Ньютона, справедливые для материальных точек, поскольку тела движутся поступательно. Трением в оси блока и трением о воздух, а также массой блока пренебрежём.
Так как нить нерастяжима и длина пружины постоянна, ускорения обоих брусков и груза равны по модулю:
\( \left| {{{\vec a}_1}} \right| = \left| {{{\vec a}_2}} \right| = \left| {{{\vec a}_3}} \right| = a \)
На рисунке показаны силы, действующие на бруски и груз.
Так как блок и нити невесомы, а трение отсутствует, то модули сил натяжения нити, действующих на груз и верхний брусок, одинаковы:
\( \left| {{{\vec T}_1}} \right| = \left| {{{\vec T}_2}} \right| = T \)
Равны по модулю и силы
\( \left| {{F_{упр2}}} \right| = \left| {{F_{упр3}}} \right| \) ,
так как пружина лёгкая.

Задание ЕГЭ по физике
Решение 1. Запишем второй закон Ньютона в проекциях на оси Ox и Oy выбранной системы координат. С учётом (1)-(3) получим:
\( Ox = M\alpha = T - {F_{тр}} \)
\( Oy = N = Mg,m\alpha = mg - T + {F_{упр}},m\alpha = mg - {F_{упр}} \)
Сложив эти уравнения, найдём ускорение тел: \( a = \frac{{2mg - {F_{тр}}}}{{M + 2m}} \)
2. Сила трения \( {F_{тр}} = \mu N = \mu Mg \)
3. Из последнего уравнения в п. 1 получим \( {F_{тр}} = \mu N = \mu Mg \)
По закону Гука \( {F_{упр}} = k\Delta l = k\left( {L - l} \right) \), тогда
\( L = l + \frac{{mMg\left( {1 + \mu } \right)}}{{k\left( {M + 2m} \right)}} = \) \( 0,1 + \frac{{0,4 \cdot 0,8 \cdot 10 \cdot \left( {1 + 0,2} \right)}}{{80 \cdot \left( {0,8 + 2 \cdot 0,4} \right)}} \) \( = 0,13 \) м.
Ответ: L = 0,13 м

P.S. Нашли ошибку в задании? Пожалуйста, сообщите о вашей находке ;)
При обращении указывайте id этого вопроса - 21078.